Search found 269 matches


St1: 30 Second movement is provided thus we can find the final quadrant = 4th. Even if we consider extremes of 1st quadrant (1 degree or 89 degrees) the resultant quadrant will still be 4. Sufficient. http://i.imgur.com/HhqhRHt.png St2: We dont know the rate; Insufficient Answer is A More can be fou...

by mevicks

Sat Jun 11, 2016 7:52 pm
Forum: Data Sufficiency
Topic: I didn't understand this question
Replies: 4
Views: 2856
by mevicks

Fri Nov 22, 2013 8:04 pm
Forum: Ask the Test Maker
Topic: GMATPrep Question Correction
Replies: 5
Views: 2968

Aren't P and Q symmetry upon y-axis? Nope they are not symmetric. We should not rely on the GMAT to provide us figures drawn to scale (implying you cannot use a protractor to measure their diagrams :D ) http://i.imgur.com/7CMYPXe.png In the figure shown, points P and Q lie on the circle with center...

by mevicks

Wed Nov 06, 2013 10:00 pm
Forum: Problem Solving
Topic: Coordinate Geometry
Replies: 7
Views: 1542

http://i.imgur.com/7CMYPXe.png In the figure shown, points P and Q lie on the circle with center O. What is the value of s? (A) 1/2 (B) 1 (C) √2 (D) √3 (E) 1/√2 10 Secs Approach: Rotate the figure at point O http://i.imgur.com/tCOjRT7.png Q has the coordinates [spoiler](1, √3) and s = 1; an...

by mevicks

Wed Nov 06, 2013 10:00 pm
Forum: Problem Solving
Topic: Coordinate Geometry
Replies: 7
Views: 1542

The Perimeter of a certain isosceles right triangle is 16+16√2. What is the length of the hypotenuse of the triangle? A)8 B)16 C)4√2 D)8√2 E)16√2 Since it is an isoceles right angled triangle its a 45-45-90 triangle with the sides in the ratio 1:1:√2 http://i.imgur.com/gMTeeyQ.png Perimet...

by mevicks

Wed Nov 06, 2013 6:27 am
Forum: Problem Solving
Topic: Relation between Perimeter and hypotenuse of triangle
Replies: 6
Views: 2845

is it to do with 5^4 conversion... but I think if we are considering even the first 6 we should reach the same answer? please correct me if wrong!

Edit : Since we have a best case scenario for first six why do should one consider the last 4?

by mevicks

Wed Nov 06, 2013 3:17 am
Forum: Problem Solving
Topic: Bombs pulled out to crack, do blast?
Replies: 9
Views: 2379

How if we only try find the probability that 2 of the last 4 bombs do not blast? The two bombs dont blast implying that 20% are always defective. Since in our case first six bombs do blast these 20% defective ones fall in the last 4. So imo we can use the P(non defective) probability for first six....

by mevicks

Wed Nov 06, 2013 3:04 am
Forum: Problem Solving
Topic: Bombs pulled out to crack, do blast?
Replies: 9
Views: 2379

Records suggest that out of every 10 Cracker Bombs manufactured by The Fluffy Fireworks, 2 do not blast when cracked. A sample of 10 Cracker Bombs manufactured by The Fluffy Fireworks is taken to crack them one by one. What is the probability that all of the first six Cracker Bombs pulled out to cr...

by mevicks

Wed Nov 06, 2013 2:47 am
Forum: Problem Solving
Topic: Bombs pulled out to crack, do blast?
Replies: 9
Views: 2379

The committee of three people is to be chosen from four married couples.What is the number of different committees that can be chosen if two people who are married to each other cannot serve on the committee? a. 16 b. 24 c. 26 d. 30 e. 32 Total people = 8 Number of selections = 3 Use the slot metho...

by mevicks

Wed Nov 06, 2013 12:08 am
Forum: Problem Solving
Topic: Please solve this real GMAT quant question
Replies: 4
Views: 4279

Q: Is n negative ? OR n < 0 ? St1: (1 - n²) < 0 n² > 1 |n| > 1 n > 1 or n < -1 n could be either negative or positive, thus INSUFFICIENT St2: n² - n - 2 < 0 (n + 1)(n - 2) < 0 Case1: (n + 1) < 0 & (n - 2) > 0 OR Case2: (n + 1) > 0 & (n - 2) < 0 Case1: n < -1 & n > 2 (This is not possi...

by mevicks

Tue Nov 05, 2013 11:52 pm
Forum: Problem Solving
Topic: Inequality
Replies: 2
Views: 1101

dddanny2006 wrote: ...
Where can I get more problems like these for practice?Any online links.
The materials provided by the GMAC are the best source for "tricky" GMAT problems... If you have exhausted a material move on to another :)

http://www.mba.com/Store/Store-catalog. ... CAT2680001

by mevicks

Tue Nov 05, 2013 11:12 pm
Forum: Data Sufficiency
Topic: Hey Guys,Please help me with this one.
Replies: 4
Views: 2030

How many members of Group G are less than 25 years of age? (1)Exactly 3/5 of the members of Group G are 25 years of age or older. (2)The 24 men in Group G constitute 30 percent of the groups membership. If everything was provided in percentages (or ratios or fractions) then we could have easily use...

by mevicks

Tue Nov 05, 2013 11:02 pm
Forum: Data Sufficiency
Topic: Hey Guys,Please help me with this one.
Replies: 4
Views: 2030

Is 720 realistic in 3 months? My $0.02 on the GMAT front... It all depends on your current mock tests score (GMATPREP results only as they are the best indicator of your probable GMAT score)... and the amount of free time you have to study for the GMAT... I'd suggest that you take a mock test to ga...

by mevicks

Tue Nov 05, 2013 5:47 am
Forum: Research MBA Programs
Topic: MBA/GMAT advice?
Replies: 2
Views: 1182

positive difference between a and b = distance between a and b on the number line OR absolute value of |a - b|

If you knew which number was bigger then you would not require the mod sign (if a > b then a - b is the positive difference)

In fact differences are always positive :)

by mevicks

Tue Nov 05, 2013 5:27 am
Forum: Problem Solving
Topic: Hey--Any Inputs would be great
Replies: 4
Views: 1313

In the rectangular coordinate system which quadrant, if any, contains no point ( x, y ) that satisfies the inequality 2x − 3y ≤ − 6 ? (A) None (B) Ι (C) ΙI (D) ΙII (E) IV Let us first trace the region depicted by 2x − 3y ≤ − 6 2x − 3y ≤ − 6 −3y ≤ -2x − 6 3y ≥ 2x + 6 ......

by mevicks

Mon Nov 04, 2013 11:04 pm
Forum: Problem Solving
Topic: PS- Coordinate Geometry
Replies: 3
Views: 3793